NSG 2525 EXAM 2: PEDIATRIC NCLEX QUESTIONS

¡Supera tus tareas y exámenes ahora con Quizwiz!

Bennett was rushed to the emergency department with possible increased intracranial pressure (ICP); which of the following is an early clinical manifestation of increased ICP in older children? (Select all that apply.) A. Macewen's sign. B. Setting sun sign. C. Papilledema. D. Diplopia

D. Diplopia

The physician has discussed the need for medication with the parents of an infant with congenital hypothyroidism. The nurse can reinforce the physician's teaching by telling the parents that: A. The medication will be needed only during times of rapid growth. B. The medication is given one time daily every other day. C. The medication schedule can be arranged to allow for drug holidays. D. The medication will be needed throughout the child's lifetime

D. The medication will be needed throughout the child's lifetime

A child is admitted to the hospital with right lower abdominal pain, anorexia, and fever. Which nursing actions are appropriate to achieve an optimum outcome for this patient?Select all that apply. A. Provide clear liquids only. B. Provide emotional support. C. Administer intravenous fluids. D. Administer IV analgesic medication. E. Administer oral antipyretic medication.

B. Provide emotional support. C. Administer intravenous fluids.

Which of the following are not causes of appendicitis? SATA A. Fecalith B. Routine usage of NSAIDs C. Infection due to Helicobacter pylori D. Lymph node enlargemnent due to viral or bacterial infection E. Diet low in fiber

B. Routine usage of NSAIDs C. infection due to Helicobacter pylori E. Diet low in fiber

What is a priority nursing consideration for a child with suspected bacterial meningitis? A. Supporting the family B. Instituting standard precautions C. Administering antibiotics as soon as possible D. Administering pain medications around the clock

C. Administering antibiotics as soon as possible

A patient is scheduled for an appendectomy at noon. while performing your morning assessment, you note that the patient has a fever of 103.8 F and rates abdominal pain at a 9 out of 10. In addition, the abdomen is distended and the patient claims that he was feeling better last night, but it seems the pain has gotten worse. The patient is having tachycardia and tachypnea. Based on the scenario, what do you suspect the patient is experiencing? A. Pulmonary embolism B. Colon Fistulae C. Peritonitis D. Hemorrhage

C. Peritonitis

The nurse is discussing long-term care with the parents of a child who has a ventriculoperitoneal shunt to correct hydrocephalus. Which information is most important for the nurse to teach the parents? A. Most usual childhood activities must be restricted B. Cognitive impairment is to be expected with hydrocephalus C. Shunt malfunction or infection requires immediate treatment D. Parental protection is essential until the child reaches adulthood

C. Shunt malfunction or infection requires immediate treatment

What most accurately describes bowel function in children born with a myelomeningocele? A. Incontinence cannot be prevented. B. Enemas and laxatives are contraindicated. C. Some degree of fecal continence can usually be achieved. D. A colostomy is usually required by the time the child reaches adolescence.

C. Some degree of fecal continence can usually be achieved.

The nurse is caring for an infant with a myelomeningocele. The parents ask the nurse why the nurse keeps measuring the baby's head circumference. Select the nurse's best response: 1. "Babies' heads are measured to ensure growth is on track." 2. "Babies with a myelomeningocele are at risk for hydrocephalus, which shows up as an increase in head size." 3. "Because your baby has an opening on the spinal cord, your infant is at risk for meningitis, which can show up as an increase in head size." 4. "Many infants with myelomeningocele have microcephaly, which can show up as a decrease in head size."

2

Which order would the nurse question for a child just admitted with the diagnosis of bacterial meningitis? 1. Maintain isolation precautions until 24 hours after receiving intravenous antibiotics. 2. Intravenous fluids at 11/2 times regular maintenance. 3. Neurological checks every hour. 4. Administer acetaminophen for temperatures higher than 38°C (100.4°F).

2.

An infant is born with a sac protruding through the spine, containing cerebrospinal fluid (CSF), a portion of the meninges, and nerve roots. This condition is referred to as: 1. Meningocele. 2. Myelomeningocele. 3. Spina bifida occulta. 4. Anencephaly.

2. Myelomeningocele.

If appendicitis were suspected, in which area of the abdomen would the nurse expect the patient to report pain? A. Upper left quadrant B. Middle left quadrant C. Upper right quadrant D. Lower right quadrant

D. Lower right quadrant

Which patients does the nurse monitor for increased status epilepticus risk?Select all that apply. A. The child who has meningitis B. The toddler exposed to lead paint C. The child with a sodium level of 112 mEq/L D. The child who fell from a second-story window E. The child with a fever of 104 degrees Fahrenheit

ABCD

The twelve-year-old patient with spina bifida exhibits learning delays. What other assessment findings does the nurse anticipate?Select all that apply. A. Slow to follow directions B. Difficulty swallowing foods C. Upper limb discoordination D. Frequent respiratory infections E. Bowel and bladder incontinence

ABCE

A child is diagnosed with early stage hydrocephalus. What actions should the nurse perform?Select all that apply. A. Place padding on all four of bed rails. B. Teach parents to expect high-pitched crying. C. Administer ondansetron (Zofran) for vomiting. D. Provide orientation to the room, call light, and personnel. E. Consult dietician for dietary supplement recommendations.

ACDE

The nurse is aware of which statement to be true regarding the incidence of testicular cancer? a. Testicular cancer is the most common cancer in men aged 30 to 50 years. b. The early symptoms of testicular cancer are pain and induration. c. Men with a history of cryptorchidism are at the greatest risk for the development of testicular cancer. d. The cure rate for testicular cancer is low.

C

You patient is 4 days post op from an appendectomy. Which assessment finding requires further evaluation? A. The patient reports their last bowel movement was the day before surgery B. The patient reports incisional pain C. The patient coughs and deep breathes while splinting the abdominal incision D. Options A and C

A. The patient reports their last bowel movement was the day before surgery

The nurse is caring for a 6-month-old infant diagnosed with meningitis. When the child is placed in the supine position and flexes his neck, the nurse notes he flexes his knees and hips. This is referred to as: 1. Brudzinski sign. 2. Cushing triad. 3. Kernig sign. 4. Nuchal rigidity.

1. Brudzinski sign.

True or False: the appendix is found on the left lower side of the abdomen and is connected to the cecum of the large intestine.

FALSE

The child with cryptorchidism is being discharged after orchiopexy, which was performed on an outpatient basis. The nurse should reinforce instructions to the parents about which priority care measure? 1.Measuring intake and output 2.Administering anticholinergics 3.Preventing infection at the surgical site 4.Applying cold, wet compresses to the surgical site

3

A nurse is giving discharge instructions to the caregivers of a patient who is taking levothyroxine (Synthroid). The nurse instruct the client to notify the physician if which of the following occurs? A. Cold intolerance B. Tremors C. Coarse, dry hair D. Muscle cramps

B. Tremors

The nurse is assigned to care for a 2-year-old child who has been admitted to the hospital for surgical correction of cryptorchidism. What is the highest priority in the postoperative plan of care for this child? 1.Force oral fluids. 2.Encourage coughing. 3.Test the urine for glucose. 4.Prevent tension on the suture.

4

The nurse is assigned to care for an infant with cryptorchidism. One testis cannot be palpated. The nurse anticipates that which diagnostic study will be prescribed to determine where the undescended testis is located in the body? 1.Cystocopy 2.Abdominal x-ray 3.Urodynamic study 4.Computed tomography scan

4

An eight-year-old child is brought to the emergency department by his parents with signs of late hydrocephalus. The nurse manages what expected findings in this patient?Select all that apply. A. Setting-sun sign B. Ongoing seizure activity C. Restlessness and irritability D. Blood pressure 140/90 mm Hg E. Heart rate of 45 beats per minute

BDE

Which is the nurse's best response to the parents of a neonate with a meningocele who ask what can they expect? 1. "After initial surgery to close the defect, most children experience no neurological dysfunction." 2. "Surgery to close the sac will be postponed until the infant has grown and has enough skin to form a graft." 3. "After the initial surgery to close the defect, the child will likely have motor and sensory deficits." 4. "After the initial surgery to close the defect, the child will likely have future problems with urinary and bowel continence."

1

The nurse is caring for a pediatric client diagnosed with eczema. Which topical medication order does the nurse anticipate for this client? 1. Corticosteroids 2. Retinoids3 . Antifungals 4. Antibacterial

1. Corticosteroids

A child diagnosed with meningitis is having a generalized tonic-clonic seizure. Which should the nurse do first? 1. Administer blow-by oxygen and call for additional help. 2. Reassure the parents that seizures are common in children with meningitis. 3. Call a code and ask the parents to leave the room. 4. Assess the child's temperature and blood pressure.

1. The child experiencing a seizure usually requires more oxygen as the seizure increases the body's metabolic rate and demand for oxygen. The seizure may also affect the child's airway, causing the child to be hypoxic. It is always appropriate to give the child blow-by oxygen immediately. The nurse should remain with the child and call for additional help.

The nurse is preparing a child for an appendectomy and notes that the child is extremely quiet. Which nursing action is correct?Select all that apply. A. Encourage the child to try to sleep. B. Tell the child that everything will be fine. C. Encourage the child to verbalize feelings. D. Provide discharge instructions to allow the parents to leave faster.

A. Encourage the child to try to sleep. C. Encourage the child to verbalize feelings.

A lumbar puncture is performed on a child suspected of having bacterial meningitis. CSF is obtained for analysis. A nurse reviews the results of the CSF analysis and determines that which of the following results would verify the diagnosis? A Cloudy CSF, decreased protein, and decreased glucose B Cloudy CSF, elevated protein, and decreased glucose C Clear CSF, elevated protein, and decreased glucose D Clear CSF, decreased pressure, and elevated protein

B Cloudy CSF, elevated protein, and decreased glucose

During the acute stage of meningitis, a 3-year-old child is restless and irritable. Which of the following would be most appropriate to institute? A Limiting conversation with the child B Keeping extraneous noise to a minimum C Allowing the child to play in the bathtub D Performing treatments quickly

B Keeping extraneous noise to a minimum

Meningitis occurs as an extension of a variety of bacterial infections due to which of the following conditions? A Congenital anatomic abnormality of the meninges B Lack of acquired resistance to the various etiologic organisms C Occlusion or narrowing of the CSF pathway D Natural affinity of the CNS to certain pathogens

B Lack of acquired resistance to the various etiologic organisms

When interviewing the parents of a 2-year-old child, a history of which of the following illnesses would lead the nurse to suspect pneumococcal meningitis? A Bladder infection B Middle ear infection C Fractured clavicle D Septic arthritis

B Middle ear infection

Which of the following activities are appropriate for a child who is recovering from orchiopexy? A. Horseback riding. B. Baseball games. C. Puzzle games. D. Running around the playground.

C

A nurse is performing an admission assessment on a newborn infant with a diagnosis of spina bifida (myelomeningocele). A priority nursing assessment for this newborn is: A. pulse rate B. palpation of the abdomen C. specific gravity of the urine D. head circumference measurement

D

A pregnant client, age 32, asks the nurse why her doctor has recommended a serum alpha fetoprotein. The nurse should explain that the doctor has recommended the test: A. Because it is a state law B. To detect cardiovascular defects C. Because of her age D. To detect neurological defects

D

A topical corticosteroid is prescribed by the HCP for a child with atopic dermatitis (eczema). Which instruction should the nurse give the parent about applying the cream? A. Apply the cream over the entire body B. Apply a thick layer of cream of affected areas only C. Avoid cleansing the area before application of the cream D. Apply a thin layer of cream and rub it into the area throughly.

D

A woman who is 6 weeks pregnant tells the nurse that she is worried her baby might have spina bifida because of a family history. What should the nurse's response be based on? A. There is no genetic basis for the defect. B. Prenatal detection is not possible yet. C. Chromosomal studies done on amniotic fluid can diagnose the defect prenatally. D. The concentration of α-fetoprotein in amniotic fluid can potentially indicate the presence of the defect prenatally.

D

The mother of a 2-month-old infant brings the child to the clinic for a well baby check. She is concerned because she feels only one testis in the scrotal sac. Which of the following statements about the undescended testis is the most accurate? A. Normally, the testes are descended by birth. B. The infant will likely require surgical intervention. C. The infant probably has only one testis. D. Normally, the testes descend by one year of age.

D

The nurse is providing discharge instructions to the parents of a 2-year-old child who had an orchiopexy to correct cryptorchidism. Which statement by the parents indicate that further teaching is necessary? A. "I'll check his temperature." B. "I'll give him medication so he'll be comfortable." C. "I'll check his voiding to be sure there's no problem." D. "I'll let him decide when to return to his play activities."

D

The nurse is taking the family history of a 2-year-old child with atopic dermatitis (eczema). Which statement by the mother is most important in formulating a plan of care for this child?Select an option, then click Submit. A."Our first child was born with a cleft lip." B."We are very careful not to get sunburns in our family." C."My first child sometimes got a diaper rash." D."My husband and our daughter are both lactose-intolerant."

D

A fifteen-year-old child presents to the emergency department with a severe head trauma, including herniation of the brain into the spinal column. The child is non-responsive and intubated. How will the nurse most correctly explain to the parents what is happening? A. "While the hydrocephalus persists, your child will not be responsive to your voice or painful stimuli." B. "Eventually your child will be able to squeeze your hand and will gradually begin to respond to touch." C. "The injury is severe, but with surgery and extended medical care in, a complete recovery can be expected." D. "Because of the type of injury sustained, the brain will not be able to maintain respiratory effort without the ventilator."

D. "Because of the type of injury sustained, the brain will not be able to maintain respiratory effort without the ventilator."

Which signs and symptoms would the nurse expect to assess in a newborn with congenital hypothyroidism? 1. Preterm, diarrhea, and tachycardia. 2. Post-term, constipation, and bradycardia. 3. High-pitched cry, colicky, and jittery. 4. Lethargy, diarrhea, and tachycardia.

2. Post-term, constipation, and bradycardia.

The nurse is aware that cloudy cerebrospinal fluid (CSF) most likely indicates: 1. Viral meningitis. 2. Bacterial meningitis. 3. No infection, as CSF is usually cloudy. 4. Sepsis.

2. The CSF in bacterial meningitis is usually cloudy.

The parents of a child being evaluated for appendicitis tell the nurse the physician said their child has a positive Rovsing sign. They ask the nurse what this means. Select the nurse's best response. 1. "Your child's physician should answer that question." 2. "A positive Rovsing sign means the child feels pain in the right side of the abdomen when the left side is palpated." 3. "A positive Rovsing sign means pain is felt when the physician removes the hand from the abdomen." 4. "A positive Rovsing sign means pain is felt in the right lower quadrant when the child coughs.

2. "A positive Rovsing sign means the child feels pain in the right side of the abdomen when the left side is palpated."

The nurse is caring for a 3-year-old who had an appendectomy 2 days ago. The child has a fever of 101.8°F (38.8°C) and breath sounds are slightly diminished in the right lower lobe. Which action is most appropriate? 1. Teach the child how to use an incentive spirometer. 2. Encourage the child to blow bubbles. 3. Obtain an order for intravenous antibiotics. 4. Obtain an order for Tylenol (acetaminophen).

2. Blowing bubbles is a developmentally appropriate way to help the preschooler take deep breaths and cough.

A parent of a newborn diagnosed with myelomeningocele asks what is a common long-term complication? The nurse's best response is: 1. Learning disabilities. 2. Urinary tract infections. 3. Hydrocephalus. 4. Decubitus ulcers and skin breakdown.

2. Urinary tract infections.

Which of the following would be included in the plan of care for a hospitalized newborn following surgical repair of a myelomeningocele. Select all that apply. 1. Skull x-rays. 2. Daily head circumference measurements. 3. MRI scan. 4. Vital signs every 6 hours. 5. Holding to breastfeed.

23

The parents of a child with meningitis and multiple seizures ask if the child will likely develop cerebral palsy (CP). Select the nurse's best response. 1. "When your child is stable, she'll undergo computed tomography (CT) and magnetic resolution imaging (MRI). The physicians will be able to let you know if she has CP." 2. "Most children do not develop CP at this late age." 3. "Your child will be closely monitored after discharge, and a developmental specialist will be able to make the diagnosis." 4. "Most children who have had complications following meningitis develop some amount of CP."

3

Which does the nurse include in a child with myelomeningocele postoperative plan of care following ligament release? 1. Encourage the child to resume a regular diet, beginning slowly with bland foods that are easily digested, such as bananas. 2. Encourage the child to blow balloons to increase deep breathing and avoid postoperative pneumonia. 3. Assist the child to change positions to avoid skin breakdown. 4. Provide education on dietary requirements to prevent obesity and skin breakdown.

3

Select the best room assignment for a newly admitted child with bacterial meningitis. 1. Semiprivate room with a roommate who also has bacterial meningitis. 2. Semiprivate room with a roommate who has bacterial meningitis but has received intravenous antibiotics for more than 24 hours. 3. Private room that is dark and quiet with minimal stimulation. 4. Private room that is bright and colorful and has developmentally appropriate activities available.

3. A quiet private room with minimal stimulation is ideal as the child with meningitis should be in a quiet environment to avoid cerebral irritation.

The nurse is planning care for a 3-month-old infant diagnosed with eczema. Which should be the focus of the nurses care for this infant? 1. Maintaining adequate nutrition 2. Keeping the baby content 3. Preventing infection of lesions 4. Applying antibiotics to lesions

3. Preventing infection of lesions

Which is included in the plan of care for a newborn who has a myelomeningocele? 1. Place the child in the prone position with a sterile dry dressing over the defect. Slowly begin oral gastric feeds to prevent the development of necrotizing enterocolitis. 2. Place the child in the prone position with a sterile dry dressing over the defect. Begin intravenous fluids to prevent dehydration. 3. Place the child in the prone position with a sterile moist dressing over the defect. Slowly begin oral gastric feeds to prevent the development of necrotizing enterocolitis. 4. Place the child in the prone position with a sterile moist dressing over the defect. Begin intravenous fluids to prevent dehydration.

4

The nurse is providing discharge instructions to the parents of a child who had an appendectomy for a ruptured appendix 5 days ago. The nurse knows that further education is required when the parent states: 1. "We will wait a few days before allowing our child to return to school." 2. "We will wait 2 weeks before allowing our child to return to sports." 3. "We will call the pediatrician's office if we notice any drainage around the wound." 4. "We will encourage our child to go for walks every day."

4. "We will encourage our child to go for walks every day."

Nurse Marianne is caring for an infant with spina bifida. Which technique is most important in recognizing possible hydrocephalus? A. Measuring head circumference. B. Obtaining skull X-ray. C. Performing a lumbar puncture. D. Magnetic resonance imaging (MRI).

A

The physician orders laboratory tests to confirm congenital hyperthyroidism in a female client with classic signs and symptoms of this disorder. Which test result would confirm the diagnosis? A No increase in the thyroid-stimulating hormone (TSH) level after 30 minutes during the TSH stimulation test. B A decreased TSH level. C An increase in the TSH level after 30 minutes during the TSH stimulation test. D Below-normal levels of serum triiodothyronine (T3) and serum thyroxine (T4) as detected by radioimmunoassay.

A No increase in the thyroid-stimulating hormone (TSH) level after 30 minutes during the TSH stimulation test.

The nurse is discussing the surgical closure of a myelomeningocele with the parents of a newborn patient. Which statement by the parents indicates the need for further teaching? A. "Surgically closing this defect will ensure my baby can walk at the right age." B. "This surgery makes me feel less like my baby is too fragile to hold and feed." C. "This surgery minimizes the problems my baby has over the course of a lifetime." D. "Surgically closing this defect reduces the chance my baby will develop infections."

A. "Surgically closing this defect will ensure my baby can walk at the right age."

The nurse is caring for a patient with suspected meningitis. Which interventions should need to be performed in caring for this patient?Select all that apply. A. Elevate head of bed to 30 degrees. B. Begin intravenous (IV) normal saline infusion. C. Administer broad-spectrum antibiotic as prescribed. D. Draw blood sample for a white blood cell (WBC) count. E. Discuss with the patient's family the importance of vaccination.

A. Elevate head of bed to 30 degrees. B. Begin intravenous (IV) normal saline infusion. C. Administer broad-spectrum antibiotic as prescribed. D. Draw blood sample for a white blood cell (WBC) count.

Analysis of cerebrospinal fluid (CSF) obtained from lumbar puncture shows bacterial meningitis. The nurse performs what action for the nine-year-old pediatric patient? A. Ensure patient is receiving antibiotics. B. Place patient on airborne precautions. C. Administer the meningococcal vaccine. D. Provide family with home care instructions.

A. Ensure patient is receiving antibiotics.

A patient is recovering after having an appendectomy. the patient is 48 hours post op from surgery and is tolerating full liquids. The physician orders for the patient to try solid foods. What types of foods should the patient incorporate in their diet? A. Foods high in fiber B. Foods low in fiber C. Foods high in carbohyrdates D. Foods low in protein

A. Foods high in fiber

While performing a physical examination on a newborn, which assessment should be reported to the physician? A. Head circumference of 40 cm. B. Chest circumference of 32 cm. C. Acrocyanosis and edema of the scalp. D. Heart rate of 160 and respirations of 40

A. Head circumference of 40 cm.

The thirteen-month-old child had prenatal microsurgery for a myelomeningocele. Which assessment finding indicates the surgery was not completely successful? A. Increased head circumference B. Child is unable to walk or stand up C. A decreased ability to pick up objects D. Child is only saying sounds like "bah bah"

A. Increased head circumference

Nurse Jeremy is evaluating a client's fluid intake and output record. Fluid intake and urine output should relate in which way? A. Fluid intake should be double the urine output. B. Fluid intake should be approximately equal to the urine output. C. Fluid intake should be half the urine output. D. Fluid intake should be inversely proportional to the urine output.

B

Which nursing diagnosis has the highest priority when planning care for an infant with eczema?Select an option, then click Submit. A.High risk for altered parenting related to feelings of inadequacy B.Altered comfort (pruritus) related to vesicular skin eruptions C.Altered health maintenance related to knowledge deficit of treatment D.Risk for impaired skin integrity related to eczema

B

the mother of Gian, a preschooler with spina bifida tells the nurse that her daughter sneezes and gets a rash when playing with brightly colored balloons, and that she recently had an allergic reaction after eating kiwifruit and bananas. The nurse would suspect that the child may have an allergy to: A. Bananas B. Latex C. Kiwifruit D. Color dyes

B

A 3-year-old patient is diagnosed with hydrocephalus. A ventricular shunt was placed to relieve the pressure. What is the nursing care priority for a patient who had a ventricular shunt put into place? A. Assessing for proper bowel movements B. Assessing neurologic status every two hours C. Monitoring for fluid leaking from the incision D. Monitoring for headaches when the patient sits up

C

After the nurse teaches the mother of a child with eczema (atopic dermatitis) how to bathe her child, Which of the following statements indicates effective teaching? A. "I let my child play in the tub for 30 mins every night" B. "My child loves the bubble bath I put in the tub." C. "When my child gets out of the tub I just pat the skin dry." D. "I make sure my child has a bath every night."

C

Spina bifida is one of the possible neural tube defects that can occur during early embryological development. Which of the following definitions most accurately describes meningocele? A. Complete exposure of spinal cord and meninges. B. Herniation of spinal cord and meninges into a sac. C. Sac formation containing meninges and spinal fluid. D. Spinal cord tumor containing nerve roots.

C

The mother accompanied her child to the clinic for a follow up after undergoing orchiopexy yesterday. Which of the following assessment findings should alert the nurse to notify the physician immediately? A. Scrotal swelling and bruising. B. Fever over 101° F. C. A green drainage from the wound. D. Discomfort or pain.

C

A nurse is planning care for a child with acute bacterial meningitis. Based on the mode of transmission of this infection, which of the following would be included in the plan of care? A No precautions are required as long as antibiotics have been started B Maintain enteric precautions C Maintain respiratory isolation precautions for at least 24 hours after the initiation of antibiotics D Maintain neutropenic precautions

C Maintain respiratory isolation precautions for at least 24 hours after the initiation of antibiotics

A child presents with anorexia, right lower quadrant pain, and nausea. Which instruction can the nurse give to the child to enhance comfort? A. "Lie on your back." B. "Lie on your abdomen." C. "Lie on your left side with knees bent." D. "Sit up straight in the chair with your legs dangling."

C. "Lie on your left side with knees bent."

A parent brings a toddler, age 19 months, to the clinic for a regular check-up. When palpating the toddler's fontanels, what should the nurse expect to find? A. Closed anterior fontanel and open posterior fontanel. B. Open anterior and fontanel and closed posterior fontanel. C. Closed anterior and posterior fontanels. D. Open anterior and posterior fontanels.

C. Closed anterior and posterior fontanels.

A child is admitted with suspected appendicitis and states that he is having pain around the umbilicus that extends into the lower part of his abdomen. In addition, he says that the pain is worst on the right lower quadrant. The patient points to his abdomen at a location which is about 1/3 distance between the anterior superior iliac spine and umbilicus. What is this area called? A. Rovsing's point B. Hamman's point C. McBurnery's point D. Murphy's point

C. McBurney's point

Ruby is receiving thyroid replacement therapy develops the flu and forgets to take her thyroid replacement medicine. The nurse understands that skipping this medication will put the client at risk for developing which of the following life-threatening complications? A.Exophthalmos B.Thyroid storm C. Myxedema coma D. Tibial myxedema

C. Myxedema coma

The nurse is working with a new graduate in developing a plan of care for a newborn infant with spina bifida (myelomeningocele) and hydrocephalus. The nurse reminds the graduate to monitor for increased intracranial pressure (ICP). Which assessment technique should be performed to detect the presence of an increase in ICP? A. Measure urine specific gravity B. Assess for increased muscle tone C. Observe anterior fontanel for bulging D. Monitor blood pressure for signs of hypotension

C. Observe anterior fontanel for bulging

The nurse assesses a two-year-old child with papilledema related to hydrocephalus. Which finding causes the nurse the most concern? A. Child is holding head and crying B. Child is lethargic, responding to voice C. Child projectile vomits when sitting up D. Child has an increased head circumference

D

The nurse is caring for a four-year-old patient diagnosed with fungal meningitis. What is the most likely etiology in this child? A. Parents refuse to vaccinate the child B. Attending a child care center during the week C. Child has been recently diagnosed with type II diabetes D. A diagnosis of acquired immunodeficiency syndrome (AIDS)

D. A diagnosis of acquired immunodeficiency syndrome (AIDS)

An eleven-year-old patient with a history of allergic rhinitis was brought to the emergency department with headache and nuchal rigidity. In addition to a lumbar puncture, for what other testing does the nurse prepare the patient? A. Lateral chest x-ray B. Urine culture for Escherichia coli C. Rectal swab for group B streptococci (GBS) D. Computed tomography (CT) scan of sinus cavities

D. Computed tomography (CT) scan of sinus cavities

Which factor(s) is/are associated with slipped capital femoral epiphysis (SCFE)?Select all that apply. 1. Obesity. 2. Female gender. 3. African descent. 4. Age of 5 to 10 years. 5. Pubertal hormonal changes. 6. Endocrine disorders.

156

What is the rationale for elevating an extremity after a soft tissue injury such as a sprained ankle? A. Elevation reduces edema formation. B. Elevation raises the child's pain threshold. C. Elevation produces deep tissue vasodilation. D. Elevation increases metabolism in the tissues.

A

What type of dehydration is defined as "dehydration that occurs in conditions in which electrolyte and water deficits are present in approximately balanced proportion"? A. Isotonic dehydration B. Hypotonic dehydration C. Hypertonic dehydration D. All types of dehydration in infants and small children

A

Which is the most common cause of diarrhea in children under 5? A. Rotavirus B. Salmonella C. Escherichia coli D. Clostridium difficile

A

Which patient below is at MOST RISK for developing acute glomerulonephritis? A. A 3 year old male who has a positive ASO titer. B. A 5 year old male who is recovering from an appendectomy. C. An 18 year old male who is diagnosed with HIV. D. A 6 year old female newly diagnosed with measles.

A

Which should be the major consideration when selecting toys for a child who is cognitively impaired? a. Safety b. Age appropriateness c. Ability to provide exercise d. Ability to teach useful skills

A

While examining a 2-year-old child, the nurse in charge sees that the anterior fontanel is open. The nurse should: A. Notify the doctor. B. Look for other signs of abuse. C. Recognize this as a normal finding. D. Ask about a family history of Tay-Sachs disease.

A. Notify the doctor.

A 2-year-old is hospitalized with suspected intussusception. Which finding is associated with intussusception? A. "Ribbon-like" stools B. "Currant jelly" stools C. Palpable mass over the flank D. Projectile vomiting

B

Which agent is associated with alteration of normal intestinal flora by antibiotics? A. Rotavirus B. Salmonella C. Escherichia coli D. Clostridium difficile

D

Which of the following electrolytes are lost as a result of vomiting? A. bicarbonate and calcium B. sodium and hydrogen C. sodium and potassium D. hydrogen and potassium

D

A 13-month-old is discharged following repair of his epispadias. Which statement made by the parents indicates they understand the discharge teaching? 1. "If a mucous plug forms in the urinary drainage tube, we will irrigate it gently to prevent a blockage." 2. "If a mucous plug forms in the urinary drainage tube, we will allow it to pass on its own because this is a sign of healing." 3. "We will make sure the dressing is loosely applied to increase the toddler's comfort." 4. "If we notice any yellow drainage, we will know that everything is healing well."

1

The nurse is assessing an infant brought to the clinic with diarrhea. He is lethargic and has dry mucous membranes. Which of the following should the nurse recognize as an early sign of dehydration? A. Tachycardia B. Bulging, tense fontanel C. Decreased blood pressure D. Capillary refill of less than 3 seconds

A

A 4-month-old is brought to the emergency department with severe dehydration. The heart rate is 198, and her blood pressure is 68/38. The infant's anterior fontanel is sunken. The nurse notes that the infant does not cry when the intravenous line is inserted. The child's parents state that she has not "held anything down" in 18 hours. The nurse obtains a finger-stick blood sugar of 94. Which would the nurse expect to do immediately? 1. Administer a bolus of normal saline. 2. Administer a bolus of D10W. 3. Administer a bolus of normal saline with 5% dextrose added to the solution. 4. Offer the child an oral rehydrating solution such as Pedialyte.

1

The nurse is caring for a child due for surgery on a Wilms tumor. The child's procedure will consist of which of the following? 1. Only the affected kidney will be removed. 2. Both the affected kidney and the other kidney will be removed in case of recurrence. 3. The mass will be removed from the affected kidney. 4. The mass will be removed from the affected kidney, and a biopsy of the tissue of the unaffected kidney will be done.

1

The nurse is caring for an infant diagnosed with Hirschsprung disease. The mother states she is pregnant with a boy and wants to know if her new baby will likely have the disorder. Select the nurse's best response. 1. "Genetics play a small role in Hirschsprung disease, so there is a chance the baby will develop it as well." 2. "There is no evidence to support a genetic link, so it is very unlikely the baby will also have it." 3. "It is rarely seen in boys, so it is not likely your new baby will have Hirschsprung disease." 4. "Hirschsprung disease is seen only in girls, so your new baby will not be at risk."

1

The nurse is caring for an infant newly diagnosed with Hirschsprung disease. What does the nurse understand about this infant's condition? 1. There is a lack of peristalsis in the large intestine and an accumulation of bowel contents, leading to abdominal distention. 2. There is excessive peristalsis throughout the intestine, resulting in abdominal distention. 3. There is a small-bowel obstruction leading to ribbon-like stools. 4. There is inflammation throughout the large intestine, leading to accumulation of intestinal contents and abdominal distention.

1

The nurse is teaching about congenital clubfoot in infants. The nurse evaluates theteaching as successful when the parent states that clubfoot is best treated when? 1. Immediately after diagnosis. 2. At age 4 to 6 months. 3. Prior to walking (age 9 to 12 months). 4. After walking is established (age 15 to 18 months).

1

The nurse receives a call from the mother of a 6-month-old who describes her child as alternately sleepy and fussy. She states that her infant vomited once this morning and had two episodes of diarrhea. The last episode contained mucus and a small amount of blood. She asks the nurse what she should do. Select the nurse's best response. 1. "Your infant will need to have some tests in the emergency room to determine if anything serious is going on." 2. "Try feeding your infant in about 30 minutes; in the event of repeat vomiting, bring the infant to the emergency room for some tests and intravenous rehydration." 3. "Many infants display these symptoms when they develop an allergy to the for- mula they are receiving; try switching to a soy-based formula." 4. "Do not worry about the blood and mucus in the stool; it is not unusual for in- fants to have blood in their stools because their intestines are more sensitive."

1

Which child is at risk for developing glomerulonephritis? 1. A 3-year-old who had impetigo 1 week ago. 2. A 5-year-old with a history of five UTIs in the previous year. 3. A 6-year-old with new-onset type 1 diabetes. 4. A 10-year-old recovering from viral pneumonia.

1

Which combination of signs is commonly associated with glomerulonephritis? 1. Massive proteinuria, hematuria, decreased urinary output, and lethargy. 2. Mild proteinuria, increased urinary output, and lethargy. 3. Mild proteinuria, hematuria, decreased urinary output, and lethargy. 4. Massive proteinuria, decreased urinary output, and hypotension.

1

Which statement by a parent is most consistent with minimal change nephrotic syndrome (MCNS)? 1. "My child missed 2 days of school last week because of a really bad cold." 2. "After camping last week, my child's legs were covered in bug bites." 3. "My child came home from school a week ago due to vomiting and stomach cramps." 4. "We have a pet turtle but no one washes their hands after playing with the turtle."

1

A toddler is hospitalized with a fractured femur. In addition to pain medication, which will best provide pain relief for this child? 1. Parents presence at the bedside 2. Age-appropriate toys 3. Deep-breathing exercises 4. Videos for the child to watch

1. Parents presence at the bedside

The nurse tells the parent that other conditions can be associated with congenital clubfoot. Select all that apply. 1. Myelomeningocele. 2. Cerebral palsy. 3. Diastrophic dwarfism. 4. Breech position in utero. 5. Prematurity. 6. Fetal alcohol syndrome.

123

Which can occur in untreated developmental dysplasia of the hip (DDH)? Select all that apply. 1. Duck gait. 2. Pain. 3. Osteoarthritis in adulthood. 4. Osteoporosis in adulthood. 5. Increased flexibility of the hip joint in adulthood.

123

A 13-year-old just returned from surgery for scoliosis. Which nursing intervention(s) is/are appropriate in the first 24 hours? Select all that apply. 1. Assess for pain. 2. Logroll to change positions. 3. Get the teen to the bathroom 12 to 24 hours after surgery. 4. Check neurological status. 5. Monitor blood pressure.

1245

Which should the nurse include in the teaching plan for a child who had surgery to correct bilateral clubfeet and had the casts removed? Select all that apply. 1. "Your child will need to wear a brace on the feet 23 hours a day for at least 2 months." 2. "Your child should see an orthopedic surgeon regularly until the age of 18 years." 3. "Your child will not be able to participate in sports that require a lot of running." 4. "Your child may have a recurrence of clubfoot in a year or more." 5. "Most children treated for clubfeet develop feet that appear and function normally." 6. "Most children treated for clubfeet require surgery at puberty."

1245

Which instruction(s) should the nurse give the parents of an adolescent with slippedcapital femoral epiphysis (SCFE)? Select all that apply. 1. Continue upper body exercises to limit loss of muscle strength. 2. Do not turn the teen in bed when complaining of pain. 3. Provide homework, computer games, and other activities to decrease boredom. 4. Do most activities of daily living for the teen. 5. Expect expressions of anger and hostility. 6. Continue setting limits on behavior.

1356

A 1-year-old child with hypospadias is scheduled for surgery to correct this condition. The nurse is asked to assist in preparing a plan of care for this child. During this developmental time period, which factor should the nurse take into account? 1.Sibling rivalry will cause regression to occur. 2.Fears of separation and mutilation are present. 3.Embarrassment of voiding irregularities is common. 4.Concern over size and function of the penis is present.

2

A 9-year-old is in a spica cast and complains of pain 1 hour after receiving intravenous opioid analgesia. What should the nurse do first? 1. Give more pain medication. 2. Perform a neuromuscular assessment. 3. Call the surgeon for orders. 4. Tell the child to wait another hour for the medication to work.

2

A male child who had surgery to correct hypospadias is seen in a primary health care provider's office for a well-baby checkup. The nurse provides instructions to the mother, knowing that which long-term complication is associated with hypospadias? 1.Infertility 2.Renal anomalies 3.Erectile dysfunction 4.Decreased urinary output

2

An 18-month-old child is being discharged after surgical repair of hypospadias. Which postoperative nursing care measure should the nurse stress to the parents as they prepare to take this child home? 1.Leave diapers off to allow the site to heal. 2.Avoid tub baths until the stent has been removed. 3.Encourage toilet training to ensure that the flow of urine is normal. 4.Restrict the fluid intake to reduce urinary output for the first few days.

2

The clinical manifestations of minimal change nephrotic syndrome (MCNS) are due to which of the following? 1. Chemical changes in the composition of albumin. 2. Increased permeability of the glomeruli. 3. Obstruction of the capillaries of the glomeruli. 4. Loss of the kidney's ability to excrete waste and concentrate urine.

2

The nurse collects a urine specimen preoperatively from a child with epispadias who is scheduled for surgical repair. The nurse reviews the child's record for the laboratory results of the urine test and would most likely expect to note which finding? 1.Hematuria 2.Bacteriuria 3.Glucosuria 4.Proteinuria

2

The parent of a child with glomerulonephritis asks the nurse why the urine is such a funny color. Which is the nurse's best response? 1. "It is not uncommon for the urine to be discolored when children are receiving steroids and blood pressure medications." 2. "There is blood in your child's urine that causes it to be tea-colored." 3. "Your child's urine is very concentrated, so it appears to be discolored." 4. "A ketogenic diet often causes the urine to be tea-colored."

2

Which finding requires immediate attention in a child with glomerulonephritis? 1. Sleeping most of the day and being very "cranky" when awake; blood pressure is 170/90. 2. Urine output is 190 mL in an 8-hour period and is the color of Coca-Cola. 3. Complaining of a severe headache and photophobia. 4. Refusing breakfast and lunch and stating he "just is not hungry."

2

which child can be discharged without further evaluation? 1. A 2-year-old who has had 24 hours of watery diarrhea that has changed to bloody diarrhea in the past 12 hours. 2. A 2-year-old who had a relapse of one diarrhea episode after restarting a normal diet. 3. A 6-year-old who has been having vomiting and diarrhea for 2 days and has decreased urine output. 4. A 10-year-old who has just returned from a Scout camping trip and has had several episodes of diarrhea

2

Which of the following is true of a Wilms tumor? Select all that apply 1. It is also referred to as neuroblastoma. 2. It can occur at any age but is seen most often between the ages of 2 and 5 years. 3. It can occur on its own or can be associated with many congenital anomalies. 4. It is a slow-growing tumor. 5. It is associated with a very poor prognosis.

2,3

A child diagnosed with a Wilms tumor is scheduled for an MRI scan of the lungs. The parent asks the nurse the reason for this test as a Wilms tumor involves the kidney, not the lung. Which is the nurse's best response? 1. "I'm not sure why your child is going for this test. I will check and get back to you." 2. "It sounds like we made a mistake. I will check and get back to you." 3. "The test is done to check to see if the disease has spread to the lungs." 4. "We want to check the lungs to make sure your child is healthy enough to tolerate surgery."

3

A child with minimal change nephrotic syndrome (MCNS) has generalized edema. The skin appears stretched, and areas of breakdown are noted over the bony promi- nences. The child has been receiving Lasix twice daily for several days. Which does the nurse expect to be included in the treatment plan to reduce edema? 1. An increase in the amount and frequency of Lasix. 2. Addition of a second diuretic, such as mannitol. 3. Administration of intravenous albumin. 4. Elimination of all fluids and sodium from the child's diet.

3

An infant is scheduled for a hypospadias and chordee repair. The parent tells the nurse, "I understand why the hypospadias repair is necessary, but do they have to fix the chordee as well?" Which is the nurse's best response? 1. "I understand your concern. Parents do not want their children to undergo extra surgery." 2. "The chordee repair is done strictly for cosmetic reasons that may affect your son as he ages." 3. "The repair is done to optimize sexual functioning when he is older." 4. "This is the best time to repair the chordee because he will be having surgery anyway."

3

The nurse has reinforced discharge instructions to the mother of an 18-month-old child following surgical repair of hypospadias. Which statement by the mother indicates a need for further teaching? 1."I should encourage fluid intake." 2."I should avoid toilet training right now." 3."I should carry my child by straddling the child on my hip." 4."I should use double diapers to hold the surgery site in place."

3

The nurse is caring for a 2-year-old child who was admitted to the pediatric unit for moderate dehydration due to vomiting and diarrhea. The child is restless, with periods of irritability. The child is afebrile with a heart rate of 148 and a blood pressure of 90/42. Baseline laboratory tests reveal the following: Na 152, Cl 119, and glucose 115. The parents state that the child has not urinated in 12 hours. After establishing a saline lock, the nurse reviews the physician's orders. Which order should the nurse question? 1. Administer a saline bolus of 10 mL/kg, which may be repeated if the child does not urinate. 2. Recheck serum electrolytes in 12 hours. 3. After the saline bolus, begin maintenance fluids of D5 1/4 NS with 10 mEq KCl/L. 4. Give clear liquid diet as tolerated.

3

The nurse is caring for a 3-month-old being evaluated for possible Hirschsprung disease. His parents call the nurse and show her his diaper containing a large amount of mucus and bloody diarrhea. The nurse notes that the infant is irritable and his abdomen appears very distended. Which should be the nurse's next action? 1. Reassure the parents that this is an expected finding and not uncommon. 2. Call a code for a potential cardiac arrest, and stay with the infant. 3. Immediately obtain all vital signs with a quick head-to-toe assessment. 4. Obtain a stool sample for occult blood.

3

The nurse is caring for a 4-year-old child with a Wilms tumor. Which of the following would the nurse expect to find on assessment? 1. Decreased blood pressure, increased temperature, and a firm mass located in oneflank area .2. Increased blood pressure, temperature within normal limits, and a firm masslocated in one flank area. 3. Increased blood pressure, temperature within normal limits, and a firm masslocated on one side or the other of the midline of the abdomen. 4. Decreased blood pressure, temperature within normal limits, and a firm masslocated on one side or the other of the midline of the abdomen.

3

The nurse is caring for a 5-month-old infant with a diagnosis of intussusception. The infant has periods of irritability during which the knees are brought to chest and the infant cries, alternating with periods of lethargy. Vital signs are stable and within age-appropriate limits. The physician elects to give an enema. The parents ask the purpose of the enema. Select the nurse's most appropriate response. 1. "The enema will confirm the diagnosis. If the test result is positive, your child will need to have surgery to correct the intussusception." 2. "The enema will confirm the diagnosis. Although very unlikely, the enema may also help fix the intussusception so that your child will not immediately need surgery." 3. "The enema will help confirm the diagnosis and has a good chance of fixing the intussusception." 4. "The enema will help confirm the diagnosis and may temporarily fix the intussus- ception. If the bowel returns to normal, there is a strong likelihood that the intussusception will recur."

3

The nurse is caring for a 9-month-old with diarrhea secondary to rotavirus. The child has not vomited and is mildly dehydrated. Which is likely to be included in the discharge teaching? 1. Administer Imodium as needed. 2. Administer Kaopectate as needed. 3. Continue breastfeeding per routine. 4. The infant may return to day care 24 hours after antibiotics have been started.

3

The nurse is caring for a newborn with hypospadias. His parents ask if circumcision is an option. Which is the nurse's best response? 1. "Circumcision is a fading practice and is now contraindicated in most children." 2. "Circumcision in children with hypospadias is recommended because it helps prevent infection." 3. "Circumcision is an option, but it cannot be done at this time." 4. "Circumcision can never be performed in a child with hypospadias."

3

The parent of a 3-week-old states that the infant was recasted this morning for clubfoot and has been crying for the past hour. Which intervention should the nurse suggest the parent do first? 1. Give pain medication. 2. Reposition the infant in the crib. 3. Check the neurocirculatory status of the foot. 4. Use a cool blow-dryer to blow into the cast to control itching.

3

The parent of a 5-year-old states that the child has been having diarrhea for 24 hours, vomited twice 2 hours ago, and now claims to be thirsty. The parent asks what to offer the child because the child is refusing Pedialyte. Select the nurse's most appropriate response. 1. "You can offer clear diet soda such as Sprite and ginger ale." 2. "Pedialyte is really the best thing for your child, who, if thirsty enough, will eventually drink it." 3. "Pedialyte is really the best thing for your child. Allow your child some choice in the way to take it by offering small amounts in a spoon, medicine cup, or syringe." 4. "It really does not matter what your child drinks as long as it is kept down. Try offering small amounts of fluids in medicine cups."

3

The parents of a child hospitalized with minimal change nephrotic syndrome (MCNS) ask why the last blood test revealed elevated lipids. Which is the nurse's best response? 1. "If your child had just eaten a fatty meal, the lipids may have been falsely elevated." 2. "It's not unusual to see elevated lipids in children because of the dietary habits of today." 3. "Since your child is losing so much protein, the liver is stimulated and makes more lipids." 4. "Your child's blood is very concentrated because of the edema, so the lipids are falsely elevated."

3

Which would the nurse expect to assess on a 3-week-old infant with developmentaldysplasia of the hip (DDH)? 1. Excessive hip abduction. 2. Femoral lengthening of an affected leg. 3. Asymmetry of gluteal and thigh folds. 4. Pain when lying prone.

3

A school-age client diagnosed with autism is admitted to the hospital because of recent vomiting and diarrhea. Which intervention by the nurse is most appropriate upon admission? 1. Take the child on a quick tour of the whole unit. 2. Take the child to the playroom immediately for arts and crafts. 3. Orient the child to the hospital room with minimal distractions. 4. Admit the child to a four-bed unit with small children

3. . Orient the child to the hospital room with minimal distractions.

The nurse is caring for a child who presents with abdominal distention and currant jelly stools. Since admission, the patient's blood pressure has decreased and the patient has become lethargic. Which is the priority nursing action for this patient? A. Notify the health care provider B. Administer intravenous (IV) pain medication C. Prepare the patient for abdominal ultrasonography D. Prepare the patient for magnetic resonance imaging (MRI) of the chest and abdomen

A

Which should be included in teaching a family about post-surgical care for slipped capital femoral epiphysis (SCFE)? Select all that apply. 1. The patient will receive help with weight-bearing ambulation 24 to 48 hours after surgery. 2. Monitoring of pain medication to prevent drug dependence. 3. Instruction on pin site care. 4. Offering low-calorie meals to encourage weight loss. 5. Correct use of crutches by the patient. 6. Outpatient physical therapy for 6 to 8 weeks.

35

A parent asks the nurse how to prevent the child from having minimal change nephrotic syndrome (MCNS) again. Which is the nurse's best response? 1. "It is very rare for a child to have a relapse after having fully recovered." 2. "Unfortunately, many children have cycles of relapses, and there is very little that can be done to prevent it." 3. "Your child is much less likely to get sick again if sodium is decreased in the diet." 4. "Try to keep your child away from sick children because relapses have been associated with infectious illnesses."

4

The nurse is caring for an 8-week-old male who has just been diagnosed with Hirschsprung disease. The parents ask what they should expect. Select the nurse's best response. 1. "It is really an easy disease to manage. Most children are placed on stool softeners to help with constipation until it resolves." 2. "A permanent stool diversion, called a colostomy, will be placed by the surgeon to bypass the narrowed area." 3. "Daily bowel irrigations will help your child maintain regular bowel habits." 4. "Although your child will require surgery, there are different ways to manage the disease, depending on how much bowel is involved."

4

The nurse receives a call from the parent of a 10-month-old who has vomited three times in the past 8 hours. The parent describes the baby as playful and wanting to drink. The parent asks the nurse what to give the child. Select the nurse's best response. 1. "Replace the next feeding with regular water, and see if that is better tolerated." 2. "Do not allow your baby to eat any solids; give half the normal formula feeding, and see if that is better tolerated." 3. "Do not let your baby eat or drink anything for 24 hours to give the stomach a chance to rest." 4. "Give your child 1/2 ounce of Pedialyte every 10 minutes. If vomiting continues, wait an hour, and then repeat what you previously gave."

4

The nurse will soon receive a 4-month-old who has been diagnosed with intussuscep- tion. The infant is described as very lethargic with the following vital signs: T 101.8°F (38.7°C), HR 181, BP 68/38. The reporting nurse states the infant's abdomen is very rigid. Which is the most appropriate action for the receiving nurse? 1. Prepare to accompany the infant to a computed tomography scan to confirm the diagnosis. 2. Prepare to accompany the infant to the radiology department for a reducing enema. 3. Prepare to start a second intravenous line to administer fluids and antibiotics. 4. Prepare to get the infant ready for immediate surgical correction.

4

The parent of a 3-year-old is shocked to hear the diagnosis of Wilms tumor and says, "How could I have missed a lump this big?" What is the nurse's best response? 1. "Do not be hard on yourself. It's easy to overlook something that has probablybeen growing for months when we see our children on a regular basis." 2. "I understand you must be very upset. Your child would have had a betterprognosis had you caught it earlier." 3. "It really takes a trained professional to recognize something like this." 4. "Do not blame yourself. This mass grows so fast that it was probably notnoticeable a few days ago."

4

Which should be obtained to make a diagnosis of slipped capital femoral epiphysis (SCFE)? 1. A history of hip trauma. 2. A physical examination of hip, thigh, and knees. 3. A complete blood count. 4. A radiographic examination of the hip.

4

Which should the nurse stress to the parents of an infant in a Pavlik harness for treatment of developmental dysplasia of the hip (DDH)? 1. Put socks on over the foot pieces of the harness to help stabilize the harness. 2. Use lotions or powder on the skin to prevent rubbing of straps. 3. Remove harness during diaper changes for ease of cleaning diaper area 4. Check under the straps at least two to three times daily for red areas.

4

The nurse is working in a pediatric surgical unit. In discussing patient-controlled analgesia (PCA) in a preoperative parental meeting, which client would be a candidate for PCA? 1. Developmentally delayed 16-year-old, postoperative bone surgery 2. 5-year-old, postoperative tonsillectomy 3. 10-year-old who has a fractured femur and concussion from a bike accident 4. 12-year-old, postoperative spinal fusion for scoliosis

4. 12-year-old, postoperative spinal fusion for scoliosis

A 10-year-old is being evaluated for possible appendicitis and complains of nausea and sharp abdominal pain in the right lower quadrant. An abdominal ultrasound is scheduled, and a blood count has been obtained. The child vomits, finds the pain relieved, and calls the nurse. Which should be the nurse's next action? 1. Cancel the ultrasound, and obtain an order for oral Zofran (ondansetron). 2. Cancel the ultrasound, and prepare to administer an intravenous bolus. 3. Prepare for the probable discharge of the patient. 4. Immediately notify the physician of the child's status.

4. The physician should be notified immediately, as a sudden change or loss of pain often indicates a perforated appendix

The parents of a newborn male with hypospadias want their child circumcised. The best response by the nurse is to inform them that: A. Circumcision is delayed so the foreskin can be used for the surgical repair. B. This procedure is contraindicated because of the permanent defect. C. There is no medical indication for performing a circumcision on any child. D. The procedure should be performed as soon as the infant is stable.

A

A 6 year old male is diagnosed with nephrotic syndrome. In your nursing care plan you will include which of the following as a nursing diagnosis for this patient? A. Risk for infection B. Deficient fluid volume C. Constipation D. Overflow urinary incontinence

A

A child with autism spectrum disorder (ASD) is admitted to the hospital with pneumonia. The nurse should plan which priority intervention when caring for the child? a. Maintain a structured routine and keep stimulation to a minimum. b. Place the child in a room with a roommate of the same age. c. Maintain frequent touch and eye contact with the child. d. Take the child frequently to the playroom to play with other children.

A

A newborn's failure to pass meconium within the first 24 hours after birth may indicate which of the following? A. Hirschsprung disease B. Celiac disease C. Intussusception D. Abdominal wall defect

A

A parent of a 6-month-old infant calls the nurse hotline and reports that an infant has had loose stools for the past 12 hours. Which question is most pertinent in the nurse's assessment? A. Can you describe the number and character of the stools? B. How much fluid has the infant consumed in the past 24 hours? C. Do you think the infant caught a bug from a family member? D. Has the infant consumed a new or different food recently?

A

A patient is nervous about having the pins used for traction removed and asks the nurse what to expect. Which statement is correct about pin removal? A. General anesthesia is used for skeletal traction removal; the patient won't be conscious during the procedure.' B. The patient will receive local anesthetics to relieve pain and the area will be draped to prevent visualization of the procedure. C. General anesthesia is used for the procedure but the patient will be conscious for immediate mobility assessment. D. Topical analgesics will be used for the procedure. The bone does not contain pain receptors so the patient feels no pain as the pin is removed

A

An 18-month-old is being discharged following hypospadias repair. Which instruction should be included in the nurse's discharge teaching? A. The child should not play on his rocking horse. B. Applying warm compresses to decrease pain. C. Diapering should be avoided for 1-2 weeks. D. The child will need a special diet to promote healing.

A

Assessment of a newborn male reveals that the infant has hypospadias. The nurse knows that: A. The infant should not be circumcised. B. Surgical correction will be done by 6 months of age. C. Surgical correction is delayed until 6 years of age. D. The infant should be circumcised to facilitate voiding

A

Parents bring their infant to the clinic, seeking treatment for vomiting and diarrhea that has lasted for 2 days. On assessment, the nurse in charge detects dry mucous membranes and lethargy. What other findings suggest a fluid volume deficit? A. A sunken fontanel B. Decreased pulse rate C. Increased blood pressure D. Low urine specific gravity

A

Parents have learned that their 6-year-old child has autism. The nurse may help the parents to cope by explaining that the child may: a. Have an extremely developed skill in a particular area. b. Outgrow the condition by early adulthood. c. Have average social skills. d. Have age-appropriate language skills.

A

A patient presents with sudden onset of gross hematuria, proteinuria and hypertension. In assessing this patient, what statement best supports the importance for the nurse to assess the presence of throat discomfort over the past two weeks? A. Streptococcal pharyngitis can induce glomerulonephritis. B. Hemolytic uremic syndrome (HUS) initially presents with pharyngitis. C. Hematuria is the primary indicator that the patient has nephrotic syndrome caused by E. coli. D. Throat discomfort is an indication of edema which is the primary cause of nephrotic syndrome.

A. Streptococcal pharyngitis can induce glomerulonephritis.

A patient with glomerulonephritis is receiving intravenous fluids to regulate hydration but begins to experience pulmonary edema and worsening hypertension. Blood serum analysis shows hyperkalemia and increased BUN. The hyperkalemia does not respond to interventions. These clinical data should be an indication for the nurse to prepare for which steps in the management of acute renal failure? A. Dialysis B. Antibiotics C. Kidney transplant D. Update vaccinations

A. dialysis

The nurse should report which urine test result to the healthcare provider? A. pH of 4 B. Absence of protein C. Absence of glucose D. Specific gravity of 1.020

A. pH of 4

A 1-year-old child presents to the clinic with an abnormal abdominal bulge. The parents state the patient has been asymptomatic and the mass has not grown in size. The mass is also immobile. Upon close observation the child also has unusual appearing eyes. Why are both an ophthalmology and renal referral necessary in this patient?Select all that apply. A. The unusual appearing eyes, with missing or partial iris, are suggestive of Wilms tumor. B. Wilms tumor develops within the eye initially. It quickly metastasizes to the kidneys as blood is filtered. C. Renal referral is required because the large, immobile, abdominal mass in such a young child is indicative of Wilms tumor. D. The asymptomatic nature of the mass suggests it is a neuroblastoma. Neuroblastoma also affects the development of the eyes. E. During development, the kidneys and the eyes undergo significant differentiation during the same time. This typically results in anomalies in the kidneys and eyes simultaneously.

AB

The nurse is caring for a pediatric patient with abnormal laboratory values for rheumatoid factor (RF), C-reactive protein (CRP), and erythrocyte sedimentation rate (ESR). Which assessment findings correspond with the abnormal laboratory results?Select all that apply. A. Enlarged knee joints B. Limping when walking C. Pain when moving joints D. Hip rotates without resistance E. Accelerated closure of the epiphyseal plates

ABC

A child is in skeletal traction. What nursing interventions will decrease the risk of infection?Select all that apply. A. Check pin sites for signs of redness and inflammation. B. Use soap and water to cleanse the area around the pins. C. Leave drainage in place to keep from disrupting the area where the pins are arranged on the skin. D. Careful examination of the pin entry sites to view any signs of loose pins. E. Remove the patient from traction for a short period of time to improve circulation.

ABD

Which assessment findings indicate to the nurse a child has Down syndrome (SATA)? a. High-arched, narrow palate b. Protruding tongue c. Long, slender fingers d. Transverse palmar creasE E Hypertonic muscle tone

ABD

Which of the following are NOT a sign and symptom of acute glomerulonephritis (poststreptococcal)? SELECT-ALL-THAT-APPLY: A. Hypotension B. Increased Glomerular filtration rate C. Cola-colored urine D. Massive proteinuria E. Elevated BUN and creatinine F. Mild swelling in the face or eyes

ABD

Both nephrotic syndrome and glomerulonephritis have similar clinical manifestations. What evidence can be used to distinguish between the two disease states?Select all that apply. A. Hypertension B. Gross hematuria C. Glomerular dysfunction D. Normal serum albumin levels E. Normal serum electrolyte levels

ABDE

The nurse develops a discharge plan for a teen who is overweight with slipped capital femoral epiphysis (SCFE). Which instructions should the nurse include in the plan?Select all that apply. A. Instruct the patient on the proper use of crutches. B. Teach the child how to perform isometric exercises. C. Instruct that patient to wear a Pavlik harness to protect the hip. D. Provide instructions regarding the use of an abduction orthosis. E. Provide information to help the patient develop good nutritional habits to reduce weight.

ABE

A nurse is making an initial home care visit for a child with muscular dystrophy. What should be part of her assessment?Select all that apply. A. Coping skills B. Presence of pets C. Height and weight D. Food in the pantry E. Width of the doorways

ACDE

A twelve-year-old child's spina bifida lesion affects the upper lumbar vertebrae. The nurse evaluates that the child is meeting therapeutic goals when the child demonstrates which behaviors?Select all that apply. A. The child participates in exercise activities daily. B. The child walks without using leg braces prescribed. C. The child has successful attempts at bladder emptying. D. The child chooses to play alone at school during recess. E. The child bathes, dresses, and puts on shoes without help.

ACE

Which set of clinical evidence will prompt a nurse to prepare for managing a patient with glomerulonephritis over nephrotic syndrome?Select all that apply. A. Hematuria B. Hypotensive C. Hypertensive D. Frothy urine E. Pallor & fatigue F. Abrupt onset edema

ACF

An adolescent presents to the urgent care clinic with a suspected fracture in the lower extremity. What are the priority assessments?Select all that apply. A. Pain assessment B. Ability to properly use crutches for ambulation C. Ability to bear weight on the affected extremity D. Sensation, color and pulse of the affected extremity E. Thorough review of pain medication that was taken prior to arriving to the clinic

AD

Which action should the nurse take to decrease the spread of infection from a patient with Clostridium difficile infection?Select all that apply. A. Enforce hand washing B. Monitor urine output C. Provide acetaminophen D. Enforce contact precautions E. Check temperature every 4 hours

AD

Autism is a complex developmental disorder. The diagnostic criteria for autism include delayed or abnormal functioning in which area(s) with onset before age 3 years (SATA)? a. Language as used in social communication b. Gross motor development c. Growth below the 5th percentile for height and weight d. Symbolic or imaginative playe. Social interaction

ADE

The charge nurse is assisting a new nurse to prepare a toddler for skin traction. What interventions will help prevent skin damage?Select all that apply. A. Use an egg-crate-type mattress for comfort. B. Use alcohol to clean areas that are prone to irritation. C. Use lotion, powder, or talc on areas prone to irritation. D. Provide a trapeze to facilitate independent repositioning. E. Applying tincture of benzoin to the intact skin that has potential for irritation.

ADE

You're providing education to a group of nursing students about nephrotic syndrome. A student describes the signs and symptoms of this condition. Which signs and symptoms verbalized by the student require you to re-educate the student about this topic? Select-all-that-apply: A. Slight proteinuria B. Hypoalbuminemia C. Edema D. Hyperlipidemia E. Tea-colored urine F. Hypertension

AEF

A child is observed limping and walking on his toes. An x-ray reveals subluxation of the femoral head. Which nursing care plan is indicated for this patient? A. Have the patient walk for 30 minutes to monitor gait. B. Provide ways to alleviate the anxiety of the diagnosis. C. Monitor neurovascular status of the affected extremity. D. Teach the patient how to walk straight without limping.

B

A child with clubfoot was treated with Ponseti casting. Following completion of the casting treatments, the patient has the final cast removed. The parents are upset because the foot appears overcorrected. Which explanation can the nurse provide regarding the rationale for the overcorrection? A. Overcorrection is necessary so that surgery will be possible. B. The foot is purposefully overcorrected and a brace will be used to prevent the recurrence of the deformity. C. Unfortunately, the overcorrection was not intended and an additional cast will be used to fix the problem. D. The overcorrection will prevent recurrence and is the last step before initiating the French physiotherapy treatment.

B

A child with gastroenteritis is receiving treatment for dehydration. Which assessment findings indicate treatment has been effective? A. Serum sodium 158 mEq/L B. Serum potassium 3.9 mEq/L C. Urine output of 100 mL in 4 hours D. Absence of skin breakdown on anus

B

A child with spina bifida has developed a latex allergy as a result of numerous bladder catheterizations and surgeries. What is the priority nursing intervention? A. Recommend allergy testing B. Provide a latex-free environment C, Use only powder-free latex gloves D. Limit the use of latex products as much as possible

B

A family brings their toddler to the clinic because of frothy urine and a swollen face. Serum analysis confirms hypoalbuminemia and elevated hematocrit levels. The child is normotensive. Based on this clinical data, what care measure should the nurse anticipate in managing this patient? A. Hemodialysis to decrease edema B. Immunosuppression with corticosteroids C. Give normal saline IV to decrease hematocrit D. Decrease protein in diet to compensate for hypoalbuminemia

B

A newborn assessment shows separated sagittal suture, oblique palpebral fissures, depressed nasal bridge, protruding tongue, and transverse palmar creases. These findings are most suggestive of: a. Microcephaly. c. Cerebral palsy. b. Down syndrome. d. Fragile X syndrome.

B

A nurse is providing a parent information regarding autism. Which statement made by the parent indicates understanding of the teaching? a. Autism is characterized by periods of remission and exacerbation. b. The onset of autism usually occurs before 3 years of age. c. Children with autism have imitation and gesturing skills. d. Autism can be treated effectively with medication.

B

A parent whose child has been diagnosed with a cognitive deficit should be counseled that intellectual impairment: a. Is usually due to a genetic defect. b. May be caused by a variety of factors. c. Is rarely due to first-trimester events. d. Is usually caused by parental intellectual impairment.

B

A patient is admitted to the hospital with complaints of nausea, vomiting, diarrhea, and severe abdominal pain. Which of the following would immediately alert the nurse that the patient has bleeding from the GI tract? A. Complete blood count. B. Guaiac test. C. Vital signs. D. Abdominal girth.

B

An 8-year-old has been found to have moderate cerebral palsy (CP). The child recently began participating in a regular classroom for part of the day. The child's mother asks the school nurse about joining the afterschool Scout troop. The nurse's response should be based on what knowledge? A. Most activities such as Scouting cannot be adapted for children with CP B. After-school activities usually result in extreme fatigue for children with CP C. Trying to participate in activities such as Scouting leads to lowered self-esteem in children with CP D. After-school activities often provide children with CP with opportunities for socialization and recreation

B

As the nurse, you know that it is important to implement a low sodium diet for a patient with nephrotic syndrome. However, it is important to implement what other type of diet due to another complication associated with this syndrome? A. Low-phosphate B. Low-fat C. High-carbohydrate D. Low-potassium

B

Dustin who was diagnosed with Hirschsprung's disease has a fever and watery explosive diarrhea. Which of the following would Nurse Joyce do first? A. Administer an antidiarrheal. B. Notify the physician immediately. C. Monitor the child every 30 minutes. D. Nothing. (These findings are common in Hirschsprung's disease.)

B

Mark, a 9-year-old with Down syndrome, is mainstreamed into a regular third-grade class for part of the school day. His mother asks the school nurse about programs such as Cub Scouts that he might join. The nurses recommendation should be based on knowing that: a. Programs such as Cub Scouts are inappropriate for children who are cognitively impaired. b. Children with Down syndrome have the same need for socialization as other children. c. Children with Down syndrome socialize better with children who have similar disabilities. d. Parents of children with Down syndrome encourage programs such as scouting because they deny that their children have disabilities.

B

Niklaus was born with hypospadias; which of the following should be avoided when a child has such condition? A. Surgery. B. Circumcision. C. Intravenous pyelography (IVP). D. Catheterization.

B

The nurse is assessing an infant with Hirschsprung's disease. The nurse can expect the infant to: A. Weigh less than expected for height and age. B. Have a scaphoid-shaped abdomen. C. Exhibit clubbing of the fingers and toes. D. Have hyperactive deep tendon reflexes.

B

The nurse is caring for an infant in the pediatric unit. The nurse observes that the infant has a weak cry and a reduced gag reflex. On obtaining the history from the mother, the nurse finds that the mother gives honey to the child on a regular basis. Which condition is likely responsible for the symptoms observed in the infant? A. Tetanus B. Botulism C. Spinal cord injury D. Guillain-Barré syndrome

B

The nurse is caring for two patients, one with a fracture that has not broken the skin and one with a fracture that has pierced the skin. The nurse will need to monitor for which complication that can occur with both fractures? A. Infection B. Hemorrhaging C. Impaired skin integrity D. Compartment syndrome

B

The nurse is educating a group of pregnant women about the importance of folic acid supplementation in the diet. Which suggestion given by the nurse is appropriate? A. "Foods rich in folic acid prevent the onset of cerebral palsy in infants." B. "Foods rich in folic acid prevent the development of neural tube defects." C. "Foods rich in folic acid prevent Guillain-Barré syndrome in infants." D. "Foods rich in folic acid prevent Werdnig-Hoffmann disease in infants."

B

The nurse receives a report on multiple patients newly admitted to the pediatric unit. Which patient should the nurse assess first? A. 5-year-old with mild muscle weakness B. 6-month-old with a fractured left femur C. 3-year-old with dislocated right shoulder D. 14-month-old with palpable anterior suture line

B

The parents of a 2-month-old girl bring her in for treatment. The infant has had infrequent bowel movements since birth, and the parents describe the recent appearance of ribbon-like-foul smelling stools when changing her diaper. What other clinical manifestation might the nurse observe? A. Abdominal bruising B. Abdominal distention C. Appropriate weight gain D. Relaxing internal sphincter

B

What type of dehydration is defined as "dehydration that occurs in conditions in which the electrolyte deficit exceeds the water deficit?" A. Isotonic dehydration B. Hypotonic dehydration C. Hypertonic dehydration D. All types of dehydration in infants and small children

B

Which activity is best suited to the 12-year-old with juvenile rheumatoid arthritis? A. Playing video games B. Swimming C. Working crossword puzzles D. Playing slow-pitch softball

B

While providing care to a pediatric patient with acute glomerulonephritis, you note the urine output to be 10 mL/hr. The patient weighs 30 lbs. As the nurse, you will want to limit what type of foods from the patient's diet? A. Calcium-rich foods B. Potassium-rich foods C. Purine -rich foods D. None of the above because the patient's urinary output is normal based on the patient's weight.

B

You're providing care to a 6 year old male patient who is receiving treatment for nephrotic syndrome. Which assessment finding below requires you to notify the physician immediately? A. Frothy, dark urine B. Redden area on the patient's left leg that is swollen and warm C. Elevated lipid level on morning labs D. Urine test results that shows proteinuria

B

A child is experiencing a fever, rigid abdomen, and is bending over, holding his right lower abdomen. What does the nurse suspect is the likely cause of this? A. Peptic ulcer B. Appendicitis C. Dehydration D. Pyloric stenosis

B. Appendicitis

A nurse receives orders that state a 9-year-old patient should be placed in traction after returning to the unit. Surgery was performed for a femoral fracture. Based on this information, which type of traction should the nurse prepare?Select all that apply. A. Skin traction B. Skeletal traction C. Continuous traction D. Intermittent traction E. Cervical (halo) traction

BC

A parent brings their 6-year-old child with Duchenne muscular dystrophy (DMD) to the primary care clinic for a well child visit. Upon assessing the patient, the nurse notes that the child displays signs of muscle weakness and difficulty walking. The nurse should ensure that the patient is using which assistive device correctly?Select all that apply. A. Hearing aids B. A back brace C. Soft, flexible soles D. Corrective lenses E. Compression socks

BC

The nurse is caring for a child with Hirschsprung disease who has abdominal distention and hard, dry stools. Which food choices would be most appropriate?Select all that apply. A. Macaroni and cheese B. Grapes and apple slices C. Boiled cabbage and broccoli D. Cinnamon raisin granola bars E. Fried chicken tenders and fries

BCD

A child in traction is frustrated because of feeling trapped. The child is experiencing constipation and says breathing feels hard at times. Which nursing interventions would benefit this patient?Select all that apply. A. Decrease fluid intake. B. Reposition the body. C. Provide a high fiber diet. D. Move the bed to play area. E. Play with bubbles or an incentive spirometer. F. Encourage more rest to decrease movement.

BCDE

During abdominal assessment, the nurse notes hypoactive bowel sounds, abdominal distention, vomiting, and currant jelly stools. Which interventions would the nurse expect to provide for this patient?Select all that apply. A. Antipyretic B. Barium enema C. IV normal saline D. Intravenous antiemetic E. Abdominal ultrasonography

BCDE

The mother of a child, who was recently diagnosed with nephrotic syndrome, asks how she can identify early signs that her child is experiencing a relapse with the condition. You would tell her to monitor the child for the following: Select-all-that-apply: A. Weight loss B. Protein in the urine using an over-the-counter kit C. Tea-colored urine D. Swelling in the legs, hands, face, or abdomen

BD

1. A 5 year old male is admitted with acute glomerulonephritis. On assessment, you note mild edema predominately in the face and tea-colored urine. The patient's blood pressure is 165/110, heart rate 95, oxygen saturation 98% on room air, and temperature 98.9 'F. In your nursing care plan, what nursing interventions will you include in this patient's plan of care? SELECT-ALL-THAT-APPLY: A. Initiate and maintain a high sodium diet daily. B. Monitor intake and output hourly. C. Encourage patient to ambulate every 2 hours while awake. D. Assess color of urine after every void. E. Weigh patient every daily on a standing scale. F. Encourage the patient to consume 4 L of fluid per day.

BDE

Which nursing actions should be implemented to ensure bowel patency is maintained for an infant with intussusception?Select all that apply. A. Restrict activity B. Perform an occult blood test C. Recommend a high-fiber diet D. Evaluate the consistency of the stool E. Take a gastrointestinal history

BDE

A child presents with diarrhea after eating at a local restaurant. Which intervention should the nurse implement first? A. Restrict fluid B. Encourage diet restriction C. Inform the health department D. Prepare droplet precaution PPW

C

A nurse is assigned to the pediatric rheumatology clinic and is assessing a child who has just been diagnosed with juvenile idiopathic arthritis. Which of the following statements about the disease is most accurate? A. The child has a poor chance of recovery without joint deformity. B. Most children progress to adult rheumatoid arthritis. C. Nonsteroidal anti-inflammatory drugs are the first choice in treatment. D. Physical activity should be minimized.

C

A nurse is preparing to perform a dressing change on a 6-year-old child with mild cognitive impairment (CI) who sustained a minor burn. Which strategy should the nurse use to prepare the child for this procedure? a. Verbally explain what will be done. b. Have the child watch a video on dressing changes. c. Demonstrate a dressing change on a doll. d. Explain the importance of keeping the burn area clean.

C

A patient who is experiencing poststreptococcal glomerulonephritis has edema mainly in the face and around the eyes. As the nurse, you know to expect the edema to be most prominent during the? A. Evening B. Afternoon C. Morning D. Bedtime

C

A preteen with no previous issues is noted to have uneven shoulders, and when performing the Adam's forward bend test the nurse observes what appears to be a rib hump. On which condition does the nurse anticipate educating the patient? A. Scheuermann's Kyphosis B. Early onset scoliosis (EOS) C. Adolescent idiopathic scoliosis D. Previously undiagnosed congenital scoliosis

C

A young child has an intelligence quotient (IQ) of 45. The nurse should document this finding as: a. within the lower limits of the range of normal intelligence. b. mild cognitive impairment but educable. c. moderate cognitive impairment but trainable. d. severe cognitive impairment and completely dependent on others for care.

C

An infant with gastroenteritis presents with severe diarrhea and vomiting. Which assessment finding is most concerning? A. Capillary refill of 3 seconds B. Serum potassium level of 4.9 mEq/L C. Two wet diapers in the last 12 hours D. Fontanels bulging when the infant cries

C

The child with Down syndrome should be evaluated for what characteristic before participating in some sports? a. Hyperflexibility b. Cutis marmorata c. Atlantoaxial instability d. Speckling of iris (Brushfield's spots)

C

The nurse is caring for a 3-month-old infant diagnosed with gastroenteritis. The parents report vomiting, diarrhea, poor skin turgor, and lethargy. Which additional finding supports the diagnosis of moderate dehydration? A. Hypothermia B. Decreased respiratory rate C. Sunken or depressed fontanel D. Urine specific gravity of 1.010

C

The nurse is discussing sexuality with the parents of an adolescent girl with moderate cognitive impairment. Which should the nurse consider when dealing with this issue? a. Sterilization is recommended for any adolescent with cognitive impairment. b. Sexual drive and interest are limited in individuals with cognitive impairment. c. Individuals with cognitive impairment need a well-defined, concrete code of sexual conduct. d. Sexual intercourse rarely occurs unless the individual with cognitive impairment is sexually abused.

C

The parents of a child with cerebral palsy ask the nurse about the advantages of ankle-foot orthoses (AFOs). What is the most appropriate response by the nurse? A. AFOs have custom seats for dependent mobilization."" B. AFOs are useful for independent mobility." C. "AFOs are used to prevent deformity and increase energy efficiency of gait." D. "AFOs provide sitting balance."

C

What should the nurse keep in mind when planning to communicate with a child who has autism? a. The child has normal verbal communication. b. The child is expected to use sign language. c. The child may exhibit monotone speech and echolalia. d. The child is not listening if she is not looking at the nurse.

C

What type of dehydration is defined as "dehydration that occurs in conditions in which water loss is in excess of electrolyte loss and is usually caused by a proportionately larger loss of water or a larger intake of electrolytes?" A. Isotonic dehydration B. Hypotonic dehydration C. Hypertonic dehydration D. All types of dehydration in infants and small children

C

When caring for a newborn with Down syndrome, the nurse should be aware that the most common congenital anomaly associated with Down syndrome is: a. Hypospadias. b. Pyloric stenosis. c. Congenital heart disease. d. Congenital hip dysplasia.

C

Which action is contraindicated when a child with Down syndrome is hospitalized? a. Determine the child's vocabulary for specific body functions. b. Assess the child's hearing and visual capabilities. c. Encourage parents to leave the child alone for extended periods of time. d. Have meals served at the child's usual mealtimes.

C

Which question would be most helpful in obtaining a nursing history from the mother of an infant with suspected intussusception? A. Is your child eating normally? B. How often has your child been vomiting? C. What do your child's stools look like? D. When did your child last urinate?

C

Which statement can the nurse use to explain to parents why nonsurgical treatments are used instead of surgical treatments when treating clubfoot? A. "The surgical method takes too long to complete." B. "Nonsurgical methods allow for a faster correction of the deformity." C. "Nonsurgical treatment will limit long-term complications and recurrence." D. "The surgical method has a limited coverage by the insurance company."

C

Will is being assessed by Nurse Lucas for possible intussusception; which of the following would be least likely to provide valuable information? A. Abdominal palpation B. Pain pattern C. Family history D. Stool inspection

C

You're collecting a urine sample on a patient who is experiencing proteinuria due to nephrotic syndrome. As the nurse, you know the urine will appear: A. Tea-colored B. Orange and frothy C. Dark and foamy D. Straw-colored

C. Dark and foamy

A patient has returned for a follow-up appointment for pharyngitis. The child now has altered electrolytes and an elevated ASO titer. The nurse notes facial edema and hypertension. Which finding confirms a diagnosis of acute poststreptococcal glomerulonephritis? A. Facial edema B. Hypertension C. Elevated ASO titer D. Altered electrolytes

C. Elevated ASO titer

Child has severe diarrhea from gastroenteritis. In which ways can the nurse determine whether the child is experiencing dehydration?Select all that apply. A. Nutritional status B. Increased sodium level C. Decreased urine output D. Tenting on the back of hand E. Child has patent skin integrity

CD

A 13-year-old patient is admitted with a fractured femur. For which signs and symptoms should the nurse evaluate the patient to identify potential systemic complications?Select all that apply. A. Arrhythmia B. Immobility C. Color changes D. Axillary petechiae E. Shallow rapid breathing

CDE

The nurse is caring for a child with Hirschsprung disease who has been diagnosed with colon inflammation. Which provider orders would the nurse anticipate?Select all that apply. A. Administer a suppository as needed. B. Provide oral fluids as desired. C. Begin preoperative checklist. D. Increase dietary fiber and fluids. E. Initiate intravenous administration of normal saline.

CE

A 5-year-old patient presents with a rib hump and visible curvature of the spine measuring 20 degrees. Which is the preferred treatment that the nurse can anticipate for this patient? A.Brace B. Surgery C. Exercise D. Observation

D

A child presents with signs of developmental dysplasia of the hip. Which statement shows the nurse understands the dysplasia? A. Dislocation has a femur that is partially in place but still stable. B. Subluxation has a femur that is fully displaced from the acetabulum. C. Dislocation has a partial displacement of the femur from the acetabulum. D. Subluxation has a partial displacement of the femur from the acetabulum.

D

A child with cerebral palsy is admitted to the hospital for corrective surgery. How should the nurse approach the subject of family participation in the child's care during hospitalization? A. By assigning the parents specific times to visit their child in the hospital B. By telling the parents to room-in and take part in all aspects of the child's care C. By scheduling nursing care when the parents are out of the child's hospital room D. By discussing the parents' desire to participate in the care of the child

D

A full-term male has hypospadias. Which statement describes hypospadias? A. The urethral opening is absent. B. The urethra opens on the dorsal side of the penis. C. The penis is shorter than usual. D. The urethral meatus opens on the underside of the penis.

D

A nurse is preparing to care for a child with a diagnosis of intussusception. The nurse reviews the child's record and expects to note which symptom of this disorder documented? A. Watery diarrhea B. Ribbon-like stools C. Profuse projectile vomiting D. Bright red blood and mucus in the stools

D

A patient diagnosed with Legg-Calvé-Perthes (LCP) disease is informed that the disease has progressed to a point where necrosis is occurring in the joint. Which statement demonstrates the nurse's knowledge of LCP progression? A. Necrosis occurs after bone is reabsorbed by the body during Stage 4. B. Necrosis occurs after the fragmentation stage and is Stage 3 of the disease. C. LCP is caused by ischemia of the head of the femur. This leads to extensive reconstitution before the onset of necrosis, stage 5. D. LCP is caused by limited blood supply to the ball of the femur. As blood flow decreases, the tissue begins to die, entering necrosis or Stage 2 of the disease.

D

An 11-year-old boy acquired a testicular malignancy at the age of 10 years. An abdominal x-ray at the age of 6 months may have revealed which disorder? A. Phimosis B. Hydrocele C. Hypospadias D. Cryptorchidism

D

An overweight adolescent complains of knee pain and limps while walking. Radiographs have been obtained to confirm the condition. The nurse would anticipate educating the patient on which condition? A. Scoliosis B. Osteomyelitis C. Legg-Calvé-Perthes disease D. Slipped Capital Femoral Epiphysis (SCFE

D

How is the treatment environment related to the anxiety levels of an infant going through casting for clubfoot? A. Making the parents leave the room helps to decrease the anxiety of the infant. B. A treatment environment with a loud noise level decreases the anxiety of the infant. C. A cold environment helps to calm the infant, which will decrease the anxiety level. D. When parental involvement is incorporated, the anxiety level of the infant is decreased.

D

The nurse is caring for a child with cerebral palsy. On electroencephalogram analysis, the nurse identifies that the child has epilepsy. Which drug should the nurse administer to manage the condition? A. Baclofen B. Reserpine C. Revodopa D. Carbamazepine

D

The nurse is caring for a child with intussusception who presented with vomiting and diarrhea. Laboratory values reveal elevated serum sodium and elevated hematocrit. Which priority action should the nurse take to achieve the expected outcomes for this patient? A. Asses the child's A1cIncrease dietary iron B. Notify the provider immediately C. Encourage the child to sit up in a chair D. Administer intravenous (IV) normal saline

D

The nurse walks into a patient's room shortly after surgical correction of intussusception and notices that the patient is very lethargic. Which nursing action is a priority? A. Assess pain B. Check vital signs C. Check urine output D. Call the rapid response team

D

The parent of a child being screened for developmental dysplasia of the hip (DDH) asks the nurse why ultrasonography, not radiography, is being used. Which statement could the nurse use to explain the reason for selecting ultrasonography? A. Ultrasonography is more sensitive because it uses low dose radiation. B. Radiography is more sensitive because it uses high intensity radiation. C. Radiography is more sensitive because it can be used to view the dislocation at any age. D. Ultrasonography is more sensitive because it allows for a penetrating look at hip abnormalities.

D

To prepare a 9-year-old child with muscular dystrophy for school, the school nurse should establish goals to help the child achieve maximum potential for normal socialization. What action will assist the school nurse in achieving these goals? A. Limiting recess activities to 30 minutes per day B. Providing the child with an assistant for all activities during school hours C. Keeping the child in a special education classroom with other children with similar disabilities D. Allowing the child to use assistive devices to move about independently through the school

D

What is the major goal of therapy for children with cerebral palsy (CP)? A. Curing the underlying defect causing the disorder B. Reversing degenerative processes that have occurred C. Preventing spread to individuals in close contact with the children D. Recognizing the disorder early and promoting optimal development

D

When assessing a child treated for intussusception, which behavioral finding would indicate the expected outcomes had been met for that child? A. Guarding B. Flexing the legs C. Crying while standing D. Knocking over blocks

D

When at home, which intervention is the most important for a child with gastroenteritis? A. Encouraging a proper diet B. Showering at least twice each day C. Washing hands with alcohol-based sanitizer D. Encouraging frequent sips of water every few minutes

D

Within the past month, the admission rate of patients with poststreptococcal glomerulonephritis has doubled on your unit. You are proving an in-service to your colleagues about this condition. Which statement is CORRECT about this condition? A. "This condition tends to present 6 months after a strep infection of the throat or skin." B. "It is important the patient consumes a diet rich in potassium based foods due to the risk of hypokalemia." C. "Patients are less likely to experience hematuria with this condition." D. "This condition is not caused by the streptococcal bacteria attacking the glomerulus, but by the immune system's response to the bacteria by creating an antigen-antibody complex which inflames the glomerulus."

D

A renal parenchymal histological report of a 2-year-old patient confirms evidence of rare bilateral Wilms tumors affecting the kidneys. As the surgeon prepares for surgery, why is palpating or putting pressure on the abdomen avoided? A. Increased pressure on the tumor may cause a significant rise in blood pressure. B. Palpation or pressure near the tumor may cause it to shift and increase the difficulty of removal. C. Only with bilateral Wilms tumors will the surgeon use extra precautions to prevent the tumors from colliding. D. Increased pressure may cause the tumor to rupture and cancer cells may spread throughout the abdominal cavity.

D. Increased pressure may cause the tumor to rupture and cancer cells may spread throughout the abdominal cavity.

A neural tube defect that is not visible externally in the lumbosacral area would be called A. meningocele. B. myelomeningocele. C. spina bifida cystica. D. spina bifida occulta.

D. spina bifida occulta.

Why is it important to understand blood proteins in a patient with nephrotic syndrome?Select all that apply. A. Hyperalbuminemia develops as edema worsens. B. Proteins accumulate in the kidney preventing normal GFR. C. Proteins are filtered in the glomerulus and lost in the urine. D. Protein metabolism decreases due to negative feedback caused by hyperalbuminemia. E. Synthesis of liver proteins cannot keep up with need and patient develops hypoalbuminemia.

DE

TRUE or FALSE: Poststreptococcal glomerulonephritis is a type of NEPHROTIC SYNDROME, which means there is the leakage of BOTH red blood cells and protein from the inflamed glomerulus into the filtrate.

FALSE

The nurse is caring for a 5-year-old who has just returned from having an appendectomy. Which is the optimal way to manage pain? 1. Intravenous morphine as needed. 2. Liquid Tylenol (acetaminophen) with codeine as needed. 3. Morphine administered through a PCA pump. 4. Intramuscular morphine as needed.

3. Morphine administered through a PCA pump offers the child control over managing pain. The PCA pump also has the benefit of offering a basal rate as well as an as-needed rate for optimal pain management.

A child is admitted with appendicitis. Which statement by the patient requires immediate nursing intervention? A. The pain hurts so much its making me nauseaus B. I have no appetite C. The pain seems to be gone now D. If i position myself on my right side, it makes the pain less severe

C. The pain seems to be gone now

A child with right lower quadrant pain and anorexia has begun vomiting. Which assessments are necessary to evaluate the outcome of nursing care for this patient?Select all that apply. A. Palpate the skin B. Auscultate the chest C. Measure urine output D. Obtain a food diary E. Measure arterial blood gases (ABGs)

A. Palpate the skin C. Measure urine output

A child is admitted with right lower quadrant abdominal pain, anorexia, and fever. The pain suddenly subsides, and the child is able to play normally but still has a fever. Which potential complication is an immediate concern for this child? A. Sepsis B. Dehydration C. Malnutrition D. Hypertension

A. Sepsis

The hospitalized child with spina bifida has broken out in a rash. What actions should the nurse take?Select all that apply. A. Place a precautions sign on the door and in the room. B. Change out the gloves in the room and outside the door. C. Request that the health care provider prescribe a steroid. D. Check the patient's vital signs for a temperature elevation. E. Ask the hospital's rapid response team to evaluate the child.

ABCD

Which of the following pathologic processes is often associated with aseptic meningitis? A Ischemic infarction of cerebral tissue B Childhood diseases of viral causation such as mumps C Brain abscesses caused by a variety of pyogenic organisms D Cerebral ventricular irritation from a traumatic brain injury

B Childhood diseases of viral causation such as mumps

Thinking back to the scenario in question 3, what other signs and symptoms are associated with appendicitis. SATA. A. Increased RBCs B. Patient has the desire to be positioned in the prone position to relieve pain C. Umbilical pain that extends in the right lower quadrant D. Abdominal flaccidity E. Abdominal rebound tenderness

C. Umbilical pain that extends in the right lower quadrant E. Abdominal rebound tenderness

Which is the best position for an 8-year-old who has just returned to the pediatric unit after an appendectomy for a ruptured appendix? 1. Semi-Fowler. 2. Prone. 3. Right side-lying. 4. Left side-lying

3. Right side-lying.

Tiffany is diagnosed with increased intracranial pressure (ICP); which of the following if stated by her parents would indicate a need for Nurse Charlie to reexplain the purpose of elevating the head of the bed at a 10 to 20-degree angle? A. Help alleviate headache. B. Increase intrathoracic pressure. C. Maintain neutral position. D. Reduce intra-abdominal pressure.

B. Increase intrathoracic pressure.

The nurse is to receive a 4-year-old from the recovery room after an appendectomy. The parents have not seen the child since surgery and ask what to expect. Select the nurse's best response. 1. "Your child will be very sleepy, have an intravenous line in the hand, and have a nasal tube to help drain the stomach. If your child needs pain medication, it will be given intravenously." 2. "Your child will be very sleepy, have an intravenous line in the hand, and have white stockings to help prevent blood clots. If your child needs pain medication, we will give it intravenously or provide a liquid to swallow." 3. "Your child will be wide awake and will have an intravenous line in the hand. If your child needs pain medication, we will give it intravenously or provide a liquid to swallow." 4. "Your child will be very sleepy and have an intravenous line in the hand. If your child needs pain medication, we will give it intravenously."

4. "Your child will be very sleepy and have an intravenous line in the hand. If your child needs pain medication, we will give it intravenously."

A child is being admitted with the diagnosis of meningitis. Select the procedure the nurse should do first: 1. Administration of intravenous antibiotics. 2. Administration of maintenance intravenous fluids. 3. Placement of a Foley catheter. 4. Send the spinal fluid and blood samples to the laboratory for cultures.

4. Cultures of spinal fluid and blood should be obtained, followed by administration of intravenous antibiotics.

A newborn has been diagnosed with hypothyroidism. In discussing the condition and treatment with the family, the nurse should emphasize: A. They can expect the child will be mentally retarded B. Administration of thyroid hormone will prevent problems. C. This rare problem is always hereditary. D. Physical growth/development will be delayed.

B. Administration of thyroid hormone will prevent problems.

The nurse cares for a ten-year-old recovering from meningitis. The nurse is most concerned about which assessment finding? A. Child reports a persistent headache. B. Child fails to respond when called by name. C. Parents report the child is not engaging in activities. D. Parents insist the child complete missed schoolwork.

B. Child fails to respond when called by name.

What are the clinical manifestations of juvenile hypothyroidism? A. Sleepiness, dry skin, diarrhea B. Dry skin, sparse hair, slowed growth C. Diarrhea, dry skin, decelerated growth D. Constipation, dry skin, enlarged thyroid

B. Dry skin, sparse hair, slowed growth

You're providing education to a group of nursing students about the care of a child with appendicitis. Which statement by a nursing student requires re-education about your teaching? A. After an appendectomy the patient may have a nasogastric tube to remove stomach fluids and swallowed air B. Non-pharmacologic techniques for a patient with appenditicitis include application of heat to the abdomen and the side-lying position C. The nurse should monitor the patient for signs and symptoms of peritonitis which includes increased heart rate, respirations, temperature, abdominal distention, and intense abdominal pain D. It is normal for some patients to have shoulder pain after a laparoscopic appendectomy

B. Non-pharmacologic techniques for a patient with appenditicitis include application of heat to the abdomen and the side-lying position

The nurse is assessing the cerebrospinal fluid (CSF) analysis for a child. Which laboratory finding helps the nurse to distinguish bacterial meningitis from viral meningitis? A. Clear cerebrospinal fluid B. Positive Gram stain C. Normal glucose content D. Normal protein content

B. Positive Gram stain

A child with spina bifida has developed a latex allergy as a result of numerous bladder catheterizations and surgeries. What is the priority nursing intervention? A. Recommend allergy testing B. Provide a latex-free environment C. Use only powder-free latex gloves D. Limit the use of latex products as much as possible

B. Provide a latex-free environment


Conjuntos de estudio relacionados

Anatomy and Physiology Chapter 6, 8, 10.1-10.4

View Set

Civil Service Exam Complete Reviewer Philippines 2017: English Vocabulary Questions

View Set

6 - Health Insurance Policy Provisions

View Set

Conjugaison arabe - forme V (augmentée)

View Set

Last Chapter end of Cram Course Review- Chapter 8 Indiana Laws and Rules Pertinent to Insurance

View Set

Unit 3 Test Managerial Accounting

View Set

Consolidated Frequent Spelling Bee Word List A - F

View Set